0 Daumen
145 Aufrufe

Aufgabe:

Bestimmen Sie, ob die folgenden Reihen absolut/bedingt konvergent oder divergent sind:

a) \( \sum\limits_{n=1}^{\infty}{(-1)^n*(√n-√(n-1))} \)

b) \( \sum\limits_{n=1}^{\infty}{(-1)^n*\frac{3^n*n!}{n^n}} \)

Avatar von

2 Antworten

+1 Daumen

a) Erweitere zur 3.binomischen Formel

b) Verwende: 3^n/n^n = (3/n)^n

Avatar von 36 k
0 Daumen

Aloha :)

Willkommen in der Mathelounge... \o/

$$S_a=\sum\limits_{n=1}^\infty(-1)^n\cdot(\sqrt{n}-\sqrt{n-1})=\sum\limits_{n=1}^\infty(-1)^n\cdot\frac{(\sqrt{n}-\sqrt{n-1})\pink{(\sqrt{n}+\sqrt{n-1})}}{\pink{(\sqrt{n}+\sqrt{n-1})}}$$$$\phantom{S_a}=\sum\limits_{n=1}^\infty(-1)^n\frac{(\sqrt n)^2-(\sqrt{n-1})^2}{\sqrt n+\sqrt{n-1}}=\sum\limits_{n=1}^\infty(-1)^n\frac{n-(n-1)}{\sqrt{n}+\sqrt{n-1}}$$$$\phantom{S_a}=\sum\limits_{n=1}^\infty(-1)^n\frac{1}{\sqrt{n}+\sqrt{n-1}}$$

Nach dem Leibnitz-Kriterium musst du für die Konvergenz der Reihe nun noch zeigen, dass die Folge$$a_n=\frac{1}{\sqrt n+\sqrt{n-1}}$$eine monotone Nullfolge ist. Die Nullfolgen-Eigenschaft ist klar, denn wenn wir den Nenner verkleinern, vergrößern wir den Bruch:$$0<a_n=\frac{1}{\sqrt n+\sqrt{n-1}}\le\frac{1}{\sqrt n}\to0$$Die Monotonie ist ebenfalss klar, denn:$$\sqrt{n+1}>\sqrt{n-1}\implies\sqrt{n+1}+\pink{\sqrt n}>\pink{\sqrt n}+\sqrt{n-1}\implies$$$$\frac{1}{\sqrt{n+1}+\sqrt n}<\frac{1}{\sqrt n+\sqrt{n-1}}\implies a_{n+1}<a_n$$

Die Reihe \(S_a\) ist daher konvergent.

Sie ist allerdings nicht absolut konvergent. Wenn du nämlich die Reihe so umordnest, dass zuerst alle positiven Summanden (also die mit geradem \(n)\) addiert werden, erhältst du eine divergente Summe:$$S^{+}_a=\sum\limits_{k=1}^\infty(-1)^{2k}\frac{1}{\sqrt{2k}+\sqrt{2k-1}}=\sum\limits_{k=1}^\infty\frac{1}{\sqrt{2k}+\sqrt{2k-1}}>\sum\limits_{k=1}^\infty\frac{1}{\sqrt{2k}+\sqrt{2k}}$$$$\phantom{S^+_a}=\frac{1}{2\sqrt2}\sum\limits_{k=1}^\infty\frac{1}{\sqrt k}>\frac{1}{2\sqrt2}\sum\limits_{k=1}^\infty\frac{1}{k}\to\infty$$

Für die zweite Summe$$S_b=\sum\limits_{n=1}^\infty(-1)^n\cdot\frac{3^nn!}{n^n}=\sum\limits_{n=1}^\infty(-1)^n\cdot b_n\quad;\quad b_n\coloneqq\frac{3^nn!}{n^n}$$gehen wir von der Reihendarstellung der Exponentialfunktion aus:$$e^x=\sum\limits_{n=0}^\infty\frac{x^n}{n!}\implies e^x\ge\frac{x^n}{n!}\stackrel{x=n}{\implies}e^n\ge\frac{n^n}{n!}\implies\frac{1}{e^n}\le\frac{n!}{n^n}$$und erhalten daraus:$$b_n=3^n\cdot\frac{n!}{n^n}\ge3^n\cdot\frac{1}{e^n}=\left(\frac3e\right)^n\ge\frac3e>1$$Die Folge \((b_n)\) ist daher keine Nullfolge, sodass die \(S_b\) divergiert.

Avatar von 148 k 🚀

Ein anderes Problem?

Stell deine Frage

Willkommen bei der Mathelounge! Stell deine Frage einfach und kostenlos

x
Made by a lovely community